LSAT and Law School Admissions Forum

Get expert LSAT preparation and law school admissions advice from PowerScore Test Preparation.

User avatar
 Dave Killoran
PowerScore Staff
  • PowerScore Staff
  • Posts: 5852
  • Joined: Mar 25, 2011
|
#46088
Complete Question Explanation
(The complete setup for this game can be found here: lsat/viewtopic.php?t=8712)

The correct answer choice is (E)

Since each answer choice contains two film buffs, the 2-4-1 fixed distribution applies. Under that distribution we have already inferred that L, V, and Y each see the Hitchcock film, and so any answer choice that contains L, V, or Y would be correct (remember, this is an Except question). Answer choice (E) contains V and Y and is therefore correct.

This game provides an excellent example of how the test makers use Numerical Distributions. Many of the questions force you to examine the templates created by each distribution and then attack the answer choices. The distributions are central to understanding this game: if you do not identify the distributions, the questions in this game cannot be answered correctly.
 hope
  • Posts: 84
  • Joined: Feb 13, 2018
|
#92316
Why can't D be correct? If we are looking for what SINGLE SOLUTION cannot be placed in Hitchcock, then G-M-Y works. Y can't go because it's separated from V. Rule 6 eliminates G. Therefore, as a SINGLE SOLUTION choice, D should be just as correct as C. Can someone help me to understand? I'm puzzled.

Preferably, Jon D because I am responding to his post on this question.
 Adam Tyson
PowerScore Staff
  • PowerScore Staff
  • Posts: 5153
  • Joined: Apr 14, 2011
|
#92320
Hey Hope, it looks to me like you actually meant to ask about question 15 rather than 19. The thread for question 15 is here:

viewtopic.php?f=317&t=8475

Answer D cannot be the COMPLETE list of those who do not see the Hitchcock film because wherever Y goes, V must also go. If Y does not see the Hitchcock film, then V would have to be included in that answer as well in order for the answer to be "complete." Otherwise, V sees the Hitchcock film without Y, breaking the rule that they stay together!

Get the most out of your LSAT Prep Plus subscription.

Analyze and track your performance with our Testing and Analytics Package.